A lifeguard is sitting in an observation chair at a pool. The lifeguard's eye level is 6.2. feet fromground. The chair is 15.4 feet from a swimmer. Calculate the measure if an angle forms when thelifeguard looks down at the swimmer.

Answers

Answer 1

We can represent this situation with a triangle, as shown below:

We need to calculate the angle that forms between the line of sight of the Life Guard and the swimmer. For that, we will have to use the tangent relation.

[tex]undefined[/tex]

A Lifeguard Is Sitting In An Observation Chair At A Pool. The Lifeguard's Eye Level Is 6.2. Feet Fromground.

Related Questions

The figure shows a quarter circle and an equilateral triangle. What is thearea of the shaded part? Give your answer to 3 significant figures. (Take it= 3.14.)7 cm

Answers

Since the triangle is equilateral, all of its interior angles have a measure of 60º.

Substract the area of the triangle from the area of a circular sector with radius 7cm enclosed by an angle of 60º to find the area of the shaded region.

The area of an equilateral triangle with side length L is:

[tex]A=\frac{\sqrt[]{3}}{4}L^2[/tex]

The area of a circular sector of radius r enclosed by an angle of θ degrees is:

[tex]A=\frac{\theta}{360}\times\pi r^2[/tex]

Replace θ=60 and r=7cm to find the area of the circular sector:

[tex]A_c=\frac{60}{360}\times3.14\times(7\operatorname{cm})^2=25.643\ldots cm^2[/tex]

Replace L=7cm to find the area of the triangle:

[tex]A_T=\frac{\sqrt[]{3}}{4}\times(7\operatorname{cm})^2=21.2176\ldots cm^2[/tex]

Then, the area of the shaded region is:

[tex]\begin{gathered} A_C-A_T=25.6433\ldots cm^2-21.2176\ldots cm^2 \\ =4.4257\ldots cm^2 \\ \approx4.43\operatorname{cm}^2 \end{gathered}[/tex]

Therefore, the area of the shaded region to 3 significant figures, is:

[tex]4.43\operatorname{cm}^2[/tex]

A submarine sandwich shop surveyed a group of 20 prospective customers surveyed would be willing to pay a maximum of $4.01 to $5
A. 20 %
B.30%
C.5%
D.10%

Answers

Answer: 10%

Step-by-step explanation:

2 out of 20 people are willing to pay a maximum of 4.01 to 5 which is 0.10 which equals 10%

i need help with my homework PLEASE CHECK WORKnumber 4

Answers

ANSWER:

Option c

[tex]h=-\frac{\operatorname{\ln}(0.62)}{0.079}[/tex]

STEP-BY-STEP EXPLANATION:

The function given in the statement is the following:

[tex]D(h)=615\cdot\:e^{-0.079h}[/tex]

If D(h) = 383, we substitute and solve for h, just like this:

[tex]\begin{gathered} 383=615\cdot \:e^{-0.079h} \\ \\ e^{-0.079h}=\frac{383}{615} \\ \\ \ln(e^{-0.079h})=\ln\left(\frac{383}{615}\right) \\ \\ -0.079h=\ln(0.62) \\ \\ h=-\frac{\ln(0.62)}{0.079} \end{gathered}[/tex]

Therefore, the correct answer is option c.

The total cost, in dollars of a membership in a fitness center is given by the function c(m) = 40m +10, where m is the number of months a person is a member. In dollars, how much is the cost of a membership for 1 year?

Answers

The cost of a membership for 1 year is 490

How to determine the cost of a membership for 1 year?

The equation of the membership is given as

c(m) = 40m + 10

From the question, we understand that:

m represents the number of months

For the cost of a membership for 1 year, the number of months is 12

i.e. m = 12

Substitute the known values in the above equation

So, we have

c(12) = 40 * 12 + 10

Evaluate

c(12) = 490

Hence, the cost is 490

Read more about linear equation at

https://brainly.com/question/4074386

#SPJ1

use the two given points and calculate the slope.(6,4),(4,-1)

Answers

EXPLANATION:

Given;

We are given two points which are shown below;

[tex]\begin{gathered} (6,4) \\ (4,-1) \end{gathered}[/tex]

Required;

We are required to calculate the slope.

Step-by-step solution;

To calculate the slope given two points, we shall use the following formula;

[tex]\begin{gathered} Slope: \\ m=\frac{y_2-y_1}{x_2-x_1} \end{gathered}[/tex]

Where the variables are;

[tex]\begin{gathered} (x_1,y_1)=(6,4) \\ (x_2,y_2)=(4,-1) \end{gathered}[/tex]

We can now substitute these values and solve;

[tex]m=\frac{-1-4}{4-6}[/tex][tex]m=\frac{-5}{-2}[/tex][tex]m=\frac{5}{2}[/tex]

Therefore.

ANSWER:

[tex]m=\frac{5}{2}[/tex]

Find the equation for the circle with a diameter whose endpoints are (8,17) and (4, - 15).

Answers

Answer:

Given that,

Circle with a diameter whose endpoints are (8,17) and (4, - 15).

To find the equation for the circle

we know that,

Equation of the circle is of the form,

[tex](x-h)^2+(y-k)^2=r^2[/tex]

where r is the radius and (h,k) is the center of the circle.

If the end points of the diameter is given, then the equation of the circle is,

[tex](x-x1)(x-x2)+(y-y1)(y-y2)=0[/tex]

Substitute the points we get,

[tex](x-8)(x-4)+(y-17)(y+15)=0[/tex]

On simplifying this we get,

[tex]x^2-8x-4x+32+y^2-17y+15y-255=0[/tex][tex]x^2+y^2-12x-2y-223=0[/tex]

Working 5 hours a day, A can Complete a work in 8 days and working 6 hours a day, B can complete the same work in 10 days. Working 8 hours a day, they can jointly complete the work in how many days?​

Answers

After solving the equation, If both A and B of them worked together, then Working 8 hours a day, they can jointly complete the work in 6 days.

What is an equation?

Two mathematical expressions' values are said to be equal in an equation, which is a statement of this fact. A mathematical formula declares that two things are equal.

The equals sign ('=') is used to indicate it.

Let the work completed be W

For A

W = 5hours = 1/8 days

1 hour =  1/8 days ÷ 5

1 hour = 1/40 days

For B

W = 6 hours = 1/10 days

1 hour = 1/60 days

Add both the equation

1 hours + 1 hours = 1/40 days + 1/60 days

2 hours = 5/120 days

2 hours = 1/24days

If both of them worked for 1 hour a day

1 hour = 1/48 days

If both of them worked for 8hour a day

8 hours = 1/48 × 8

             = 1/6 days

Thus, if both of them worked together, then Working 8 hours a day, they can jointly complete the work in 6 days.

Learn more about equations

https://brainly.com/question/2972832

#SPJ9

HELP PLEASEEEEE!!!!!!

Answers

One rational number between -0.45 and -0.46 is -0.451, such that:

-0.45 > -0.451 > -0.46

How to find a rational number in the given interval?

So we want to find a rational number between -0.45 and -0.46. Remember that any decimal number with a finite number of digits after the decimal point (like the above numbers) is a rational number.

So:

3.16436

2.21412

1.4

All of these are rational numbers.

Then a rational number on the interval -0.45 and -0.46 could be -0.451, which is smaller than -0.45 and larger than -0.46

Learn more about rational numbers:

https://brainly.com/question/12088221

#SPJ1

I need help with this practice,I will send you an additional pic that goes along with this problem Freya went to her local park to find 5 organisms/species She found 5 and wrote down the name of these organisms and the quantity of each she seen:Eastern gray squirrels/14 individualsWolf spiders/2 individualsPaper wasps/9 individualsBlack vulture/1 individualNorthern cardinals/7 individuals

Answers

ANSWER :

The answer is 5/33 or 0.15

EXPLANATION :

From the problem, we have a total of 5 number of species.

Using the given Biodiversity Index formula :

[tex]\frac{\text{ total number of species}}{\text{ total number of individuals}}=\frac{5}{14+2+9+1+7}=\frac{5}{33}\quad or\quad0.1515[/tex]

In how many ways can a committee of 5 be chosen from 9 people given that Jones must be one of them?

Answers

There are 126 ways to choose a committee of five from a group of nine people using combinations; one of them must include JONES.

What do we mean by COMBINATIONS?Combinations are a mathematical method for calculating the number of alternative arrangements. In a collection of objects where the order of the selection is irrelevant. You are free to choose any combination of the available things.Mathematically it can be expressed as : nCr = n! / r!(n-r)!

So, we have a comiitte of 5 be chosen from 9 people and JONES must be one of them -

Using the rule of combinantion - nCr = n! / r!(n-r)!We have n = 9 and r= 59C5 = 9! / 5!(9-5)!C(9,5) = 3024/24C(9,5) = 126

Therefore, there are a total of 126 ways in which we can decide that JONES must be on it.

Know more about combinations here:

https://brainly.com/question/28065038

#SPJ9

I'll send a picture! please answer fast!

Answers

Given data:

The given expression for the points is,

[tex]10w-3t+5[/tex]

Thus, the expression or

10. The product of 6 and a number when added to 5 is equal to 1 less than 9 times the number.

Answers

6x+5=9x-1
6x+4=9x
3x=4
X=4/3

Answer:

x = 2

Step-by-step explanation:

Hello!

Let the unknown number be x. The product of 6 and that number can be represented by 6x. And since we are adding 5, we can use 6x + 5 to represent that expression.

This equation is equal to the difference of 9x and 1, or 9x - 1.

Solve for x6x + 5 = 9x - 15 = 3x - 16 = 2x2 = x

The value of x is 2.

while digging in his garden , will pushes a shovel into the ground at an 80 degree angle with 585 newtons of force . show the resolution of the force into its retangular components

Answers

Solution

Part a

Part b

For this case we can do this:

Fx= 585 N* cos 80= 101.58 N

Fy= 585N * sin 80= 576.11 N

Then the best answer is:

B. (102, 576) N

Identifying the Type of Series
3+6+12+ 24 + ...

5+7+10+14+

1+2+3+4+

2+4+2+4+

Answers

1) So look closely at first, you’ll see that each successive term is the twice of preceding term
Thus we get 48 and then 96 and so on
2) we add 2 to the first term to get 7 then we add 3 to get 10 and then 4 to get 14
Thus now we add 5 to get 19 and so on
3) this question speaks for itself
4) it’s just repeating with 2 4 2 4 2 4 and so on

Answer:

geometric, neither, arithmetic, and neither

Step-by-step explanation:

right on edg 2023

While solving an equation 2x^2+32=0 the answer calculated is x=+-4i I understand the +- means the answer can be positive or negative but what does the i mean

Answers

Answer:

Explanation:

Given:

[tex]\begin{gathered} 2x^2+32=0 \\ \text{The answer is x=}+-4i \end{gathered}[/tex]

To fully understand how we get the given answer, we simplify the equation first:

[tex]\begin{gathered} 2x^2+32=0 \\ \text{Simplify and rearrange} \\ 2x^2=-32 \\ x^2=-\frac{32}{2} \\ x^2=-16 \\ \end{gathered}[/tex]

Next, we apply the rule:

[tex]\begin{gathered} \text{For x}^2=f(a),\text{ the solutions are } \\ x=\sqrt[]{f(a)} \\ x=-\sqrt[]{f(a)} \end{gathered}[/tex]

So,

[tex]\begin{gathered} x^2=-16 \\ x=\sqrt[]{-16},x=-\sqrt[]{-16} \end{gathered}[/tex]

Then, we also apply the radical rule:

[tex]\begin{gathered} \sqrt[]{-a}=\sqrt[]{-1}\sqrt[]{a} \\ So, \\ x=\sqrt[]{-16} \\ =\sqrt[]{-1}\sqrt[]{16} \\ \text{Then, apply the imaginary number rule:} \\ \sqrt[]{-1}=i \\ \text{Hence,} \\ x=4i \end{gathered}[/tex]

For

[tex]\begin{gathered} x=-\sqrt[]{-16} \\ Use\text{ the same steps} \\ x=-4i \end{gathered}[/tex]

Therefore the x-values are: x=4i, x=-4i. The i on the answer means imaginary number. It is a number that, when squared, has a negative result.

factorise 10y²+21y-10​

Answers

Answer:

(2y+5)x(5y-2)

Step-by-step explanation:

For a polynomial of the form ax² - bx + c rewrite the middle term as a sum of two terms whose product is a·c=10·-10 = -100 and whose sum is b= 21.

We factor 21 out of 21y.

[tex]\boxed{\large\displaystyle\text{$\begin{gathered}\sf \bf{10y^{2}+21(y)-10 } \end{gathered}$}}[/tex]

Rewrite 21 as −4 plus 25.

[tex]\boxed{\large\displaystyle\text{$\begin{gathered}\sf \bf{10y^{2}+(-4+25)y-10 } \end{gathered}$}}[/tex]

Apply the distributive property.

[tex]\boxed{\large\displaystyle\text{$\begin{gathered}\sf \bf{10y^{2}-4y+25y-10 } \end{gathered}$}}[/tex]

Factor the highest common denominator of each group. Group the first two terms and the last two.

[tex]\boxed{\large\displaystyle\text{$\begin{gathered}\sf \bf{(10y^{2}-4y)+25y-10 } \end{gathered}$}}[/tex]

Factor the highest common denominator (GCF) of each group.

[tex]\boxed{\large\displaystyle\text{$\begin{gathered}\sf \bf{2y(5y-2)+5(5y-2) } \end{gathered}$}}[/tex]

Factor the polynomial by factoring the greatest common denominator, 5y-2.

[tex]\boxed{\boxed{\large\displaystyle\text{$\begin{gathered}\sf \bf{(5y-2)(2y+5) } \end{gathered}$}}}[/tex]

The answer is: (5y - 2)(2y + 5)

If sin 0= -3/5 in quadrant 3, what is cos 0?

Answers

Given

[tex]sin\theta=-\frac{3}{4}[/tex]

Solution

Recall : SOHCAHTOA

The final answer

Option A

[tex]Cos\text{ }\theta=-\frac{4}{5}[/tex]

Use the ALEKS calculator to write as a percentage.
31
32
Round your answer to the nearest tenth of a percent.
0%
X 5
?

Answers

Divide 31 into 32 and then multiply the result by 100 to get the percent:

[tex]\begin{gathered} \frac{31}{32}=0.96875 \\ \\ 0.96875*100=96.875 \end{gathered}[/tex]Then, to the nearest tenth of a percent the given fraction is 96.9%

How many solutions does the system of equations have? Explain1. Y = 5x - 4Y = -4 + 5xHow many solutions does the system of equations have? Explain2.Y=2x+3Y=-3x+6How many solutions does the system of equations have? Explain3. Y=-4x+5Y=-4x+5How many solutions does the system of equations have? Explain

Answers

1. it has infinite solutions because since the equations are equal it is a line equation and a line has infinite points

2. It will have only one solutions, because the lines aren't parallel or the same line (we know that because there won't be a number that you can multiply by one of the equations and obtain the other one)

3.it has infinite solutions because since the equations are equal it is a line equation and a line has infinite points

Travelers arriving at Cape Town International Airport 70% of the travelers fly on major airlines, 20% by on privately owned planes and the remainder by on commercially owned planes not belonging to a major airline. Of those traveling on major airlines, 40 degrees are traveling for business reasons, whereas 70% of those arriving on private planes and 80% of those arriving on other commercially owned planes are traveling for business reasons Suppose that we randomly select one person arriving at this airport. What is the probability that the traveler is flying privately for business reasons?

Answers

Step 1

Given;

[tex]\begin{gathered} 20\text{\% fly privately owned jets} \\ 70\text{\% of those arriving on the plane are travelling for business reason} \end{gathered}[/tex]

Kali just started a new sales floor job to save for college. She earns 15.75 plus a flat fee of 50 . She wants to earn between 200 and 400 . The following inequality represents her earning potential

200 ≤ 15.75x + 50 ≤ 400 Solve the inequality PLEASE HELP ASAP
!!

Answers

Kali needs to work between 10 to 22 hours to earn an income between 200 and 400

Flat fee earned by Kali = 50

Earning per hour of Kali = 15.75

Required income is between 200 and 400

Inequality: Mathematical expressions with inequalities are those in which the two sides are not equal. Contrary to equations, we compare two values in inequality. Less than (or less than or equal to), greater than (or greater than or equal to), or not equal to signs are used in place of the equal sign.

Inequality represents the equation:

200<=15.75x + 50<=400

200<= 15.75x + 50

150<= 15.75x

x = 9.52

So, she needs to work a minimum of 10 hours

15.75x+50<=400

15.75x <=350

x <= 22.22

So, she can work a maximum of 22 hours

Learn more about inequality:

https://brainly.com/question/20383699

#SPJ1

After a 35% reduction, you purchase a new bike for $282.75. What was the price of the bike before the reduction?

A) First write an equation you can use to answer this question. Use x as your variable and express any percents in decimal form in the equation.

B) Solve your equation in part [A] to find the original price of the bike.

Answers

The equation to represent this situation is given by 0.65x = 282.75 .

In the various mathematical formulas that are used, the equals sign is used to show that two expressions on either side of the equal sign are equal.  The meaning of the word equation and its cognates might vary slightly depending on the language.

Finding the exact values of the unknown variables that result in the given equality is the very first step in the solving of a variable equation. The values of the unknown variables that fulfil the equality are the equation's solutions, also known as the variables for which the equation must be solved. There are two sorts of equations.

Le t the original cost of the bike be x dollars.

After 35% reduction the new cost is x - (35% of x) = 0.65 x

Therefore 0.65 x = 282.75

or, x = 435

Therefore the cost of the bike is $435.

To learn more about equation visit:

https://brainly.com/question/10413253

#SPJ1

URGENT!! ILL GIVE
BRAINLIEST!!!!! AND 100
POINTS!!!!!

Answers

The correct option b: 49° because the angles are supplementary to angle d.

What is defined as supplementary angles?The definition of supplementary is linked to angles that form a straight angle when joined together. When two angles add up to 180 degrees, they are referred to as supplementary angles. If two angles are supplementary. One of its angles is just an acute angle, while another is an obtuse angle.

For the given question.

∠d = 131 degrees.

∠d + ∠f = 180  (supplementary angle)

∠f = 180 - 131

∠f = 49 degrees

Now,

∠f = ∠g = 49 degrees (vertically opposite angles)

∠f = ∠c = 49 degrees (alternate interior angles)

Thus, the angle d is supplementary to angle f, c and g.

To know more about the supplementary angles, here

https://brainly.com/question/12919120

#SPJ13

Granny Smith is making strawberry jelly. She places 6.3 ounces in each jar to give as gifts.granny was able to fill 9.5 jars. How many ounces of strawberry jelly did granny make

Answers

6.3 x 9.5 = 59.85 ounces of jelly

Solve the following. List all possible possible solutions for the ambiguous case. #7

Answers

The sum of the interior angles of any triangle is always 180º:

[tex]A+B+C=180[/tex]

Use the equation above and the given data to find C:

[tex]\begin{gathered} C=180º-A-B \\ C=180º-38º-72º \\ C=70º \end{gathered}[/tex]

Law of sines:

[tex]\frac{a}{sinA}=\frac{b}{sinB}=\frac{c}{sinC}[/tex]

Use the pair of ratios for a and b to solve a:

[tex]\begin{gathered} \frac{a}{sinA}=\frac{b}{sinB} \\ \\ a=sinA*\frac{b}{sinB} \\ \\ a=sin38º*\frac{12}{sin72º} \\ \\ a=7.8 \end{gathered}[/tex]

Use the pair of ratios for b and c to solve c:

[tex]\begin{gathered} \frac{c}{sinC}=\frac{b}{sinB} \\ \\ c=sinC*\frac{b}{sinB} \\ \\ c=sin70º*\frac{12}{sin72º} \\ \\ c=11.9 \end{gathered}[/tex]Thenm, the solution for the given triangle is:A=38ºB=72ºC=70ºa=7.8b=12c=11.9

Jamar finds some nickels and quarters in his change purse. How much money (in cents) does he have if he has 4 nickels and 11 quarters? How much money (in cents) does he have if he has n nickels and q quarters?

Answers

1 nickel is worth 5 cents and 1 quarter is worth 25 cents.

So, if he has 4 nickels, this is worth 4 times 5 cents and if he has 11 quarters, this is worth 11 times 25 cents.

Both are worth the sum of these, so:

[tex]4\cdot5+11\cdot25=20+275=295[/tex]

So, he has 295 cents.

If he has n nickels and q quarters, he has 5 times n plus 25 times q worth, so he has:

[tex]5n+25q[/tex]

Fine the end behavior and x-value of holeEquation is to the right

Answers

Given the function:

[tex]\: f\mleft(x\mright)=\frac{x+1}{\left(x-3\right)\left(x^2-1\right)}[/tex]

The end behaviour of the function f(x) describes the behaviour of the function as x approaches +∞ and -∞.

When x approaches +∞,

[tex]\lim _{x\rightarrow\infty}f(x)=\text{ }\lim _{x\rightarrow\infty}\frac{x+1}{(x-3)(x^2-1)}=0[/tex]

Thus, as x approaches +∞, the function f(x) approaches zero.

When x approaches -∞,

[tex]\lim _{x\rightarrow-\infty}f(x)=\text{ }\lim _{x\rightarrow-\infty}\frac{x+1}{(x-3)(x^2-1)}=0[/tex]

Thus, as x approaches -∞, the function f(x) approaches zero.

x-value of the hole:

For a rational function f(x) given as

[tex]f(x)=\frac{p(x)}{q(x)}[/tex]

Provided that p(x) and q(x) have a common factor (x-a), the function f(x) will have a hole at x=a.

Thus, from the function f(x)

[tex]f(x)=\frac{x+1}{(x-3)(x^2-1)}[/tex]

by expansion, we have

[tex]f(x)=\frac{x+1}{(x-3)(x^{}-1)(x+1)}[/tex]

The expression (x-1) is a common factor of the numerator and the denominator.

Thus,

[tex]\begin{gathered} x-1=0 \\ \Rightarrow x=1 \end{gathered}[/tex]

Hence, the x-value of the hole is 1.

Save-A-Lot Bank is advertising a rate of 2.5% interest compounded annually.If $2000 is invested, how much money, to the nearest cent, will be inthe account after 10 years.

Answers

Question on Compound Interest.

The formula below can be used to calculate the compound interest;

[tex]\begin{gathered} A\text{ = p(1+}\frac{r}{100})^n \\ \text{Where A = amount,(\$) (that is, the money that will be in the account)} \\ r=\text{interest rate per annum, (\%)} \\ P=Pr\text{incipal, (\$), ( that is, the money invested)} \\ n=\text{ number of periods, years, } \end{gathered}[/tex]

Where A= ? , P =$2000, r =2.5% and n = 10 years

Substituting these values into the formula above, we get

Note that: Amount = Principal + Interest, though not needed in this question.

[tex]\begin{gathered} A=P(1+\frac{r}{100})^n_{} \\ \\ A=2000(1+\frac{2.5}{100})^{10} \\ \\ A=2000(1+0.025)^{10} \\ A=2000(1.025)^{10}\text{ }=\text{ 2560.169 }\approx\text{ \$2560.17} \end{gathered}[/tex]

Thus, the correct answer is $2560.17

Given the formula FV = P + Prt, what is the future value of a savings account that had an initial deposit of $7,900 earning 6.5% simple interest for 4 years?$10,074.23$9,954.00$9,376.85$10.756.43None of these choices are correct.

Answers

Step 1: Use the formula below to find the future value:

Fv = P + Prt

P = present value or initial deposit

r = rate in %

t = time in years

Step 2: List the given data

P = $7900

r = 6.5% = 0.065

t = 4 years

Step 3: Substitute the values of P, r and t to find the future value.

FV = P + Prt

= 7900 + 7900 x 0.065 x 4

= 7900 + 2054

= $9954.00

Stop 4: Final answer

Future value = $9954.00

Given the general form: F( x )= a(x^2)+bx+cConvert it to vertex form (also known as standard form) by putting the values for a,h and k into the correct boxes.F(x)=a(x-h)^2+kIdentify the vertex(x,y)General form: F( x )=1 x^2+6 x +-1 Vertex form: F( x )= Answer for part 1 and coordinate 1 (x- Answer for part 1 and coordinate 2 )^2 +Answer for part 1 and coordinate 3Vertex: (Answer for part 2 and coordinate 1,Answer for part 2 and coordinate 2)

Answers

[tex]\begin{gathered} Vertex\colon\: (-3,-10) \\ Vertex\: form\: equation\colon\: f(x)=(x+3)^2-10 \end{gathered}[/tex]

1) In order to convert from the standard version to the vertex form we'll need to find the vertex of that parabola:

[tex]f(x)=x^2+6x-1[/tex]

We can find the vertex, using these formulas:

[tex]\begin{gathered} x=h=-\frac{b}{2a}=\frac{-6}{2}=-3 \\ k=(-3)^2+6(-3)-1=9-18-1=9-19=-10 \\ V(-3,-10) \end{gathered}[/tex]

So this is the vertex of that parabola at point (-3,-10)

2) Now, note that the coefficient a = 1, and with the vertex, we can now rewrite that equation into the vertex form:

[tex]\begin{gathered} y=a(x-h)^2+k \\ y=(x-(-3))^2+(-10) \\ y=(x+3)^2-10 \end{gathered}[/tex]

Other Questions
Miranda did a survey of what time peopleget to their office on Monday morning.The table shows her results for150 people.Timebefore 7 am7am to 7:59 am8am to 8:59 am9am to 9:59 amNo of people15397521a) What percentage of the people surveyed arrive at their office before 7 am?Optional working+Answer: b) What percentage of the people surveyed arrive at their office after 7:59 am?Optional workingAnswer:% distinguish between kinetic energy and potential energy Choose the correct completion. Yo _____ en el agua. Question 1 options: a) esqui b) esquiaste c) esquiaron d) esqui which statement is true of this function f x equals 1/5 - 2 (2580+2920+x)/3=3000 A figure is rotated 270 degrees counterclockwise about the origin. Then the image is rotated 90 degrees counterclockwise about the origin. Complete the coordinate notation that represents the series of rotations. What can you conclude about the position of the figure after the series of rotations?If you don't mind, I need the answer ASAP Consider the following system of equations 4x + 6=24 and 2x + 3y=8 How many solutions does this system have? Justify your answer. Part B If we triple each side of the first equation 4x+6y=24, we have 12x+18y=72. Explain why the new system 12x+18y=72 and 2x+3y=8 has the same number of solutions as the original system. In which of these models dothe particles have the LEASTamount of energy?A. Model A (a solid)B. Model C (a gas)C. Model B (a liquid) Now that youve toured the world of prehistoric and ancient art, summarize your findings. Art preserves various aspects of the lives of people who lived long ago, including those from prehistoric and ancient times. What are your favorite aspects of life that have been preserved in prehistoric and ancient art? Please share your thoughts in at least three thoughtful, well-constructed sentences. What percent of the millionaires surveyed stick to the budgets they create? How is thisa deciding action? Please show steps 1/2 divided by 1/2 Is a number closer to 0 greater or less than a number more away from 0? (negatives and positives) The measure of an angle is 78.3. What is the measure of its complementary angle? A perfect score on a test with 25 questions is 100. Each question is worth 4 points. Fred got a score of 68 on the test to write a division sentence using negative numbers where the quotient represents the number of questions Fred answered incorrectly how was mexican-petit gulf seed developed? Im needing guidance solving this equation 9x-9=3x+7 the equation of three lines are given below line 1:y=5/3x-6line 2:10x-6y=8line 3: 3y=5x+7for each pair of lines determine whether they are parallel perpendicular or neither line1 and line 2: parallel perpendicular or neither Line 1 and line 3:parallel perpendicular or neither line 2 to a line 3:parallel perpendicular or neither the number of mole of iron produced from 0.216 mole of aluminum is:3 FeO (l) + 2 Al (l) 3 Fe (l) + 1 Al2O3 When you drop a .44 kg apple earth exerts a force on it that accelerates it at 9.8 m/s^2 towards the earth's surface. newtons third law, the apple must exert an equal but opposite force on earth. if the mass of earth is 5.98 x 10 ^24 what is the magnitude of the earths acceleration towards the apple USChoose the definition for the function.1(-x + 2 x < 1 --x+ 2 x > 1C. Y = { x + 1x S1a.y=x+1--x+ 2 x51 d. y = { x+1b. y = x + 1-x + 2 x 2 16*x > 1Enter